What is the measure of ∠ A if it is congruent to ∠ B?

Answers

Answer 1

If measure of angle A is congruent to measure of angle B, then angle will be equal to angle B.

Define the term congruent angles?

Angles that really are identical to each other are referred to as congruent angles (and to themselves).

Angles that are congruent can also be acute, obtuse, exterior, and interior. No matter what kind of angle you have, if indeed the measure for angle one and angle two are the same, the angles are congruent.Two or more angles must have equal measurements in degrees as well as radians to be considered congruent. Congruent angles don't have to be created with the same figures or face the same direction (rays, lines, or the line segments). Both angles are congruent if the two measurements of the angles are equivalent.

Thus, if measure of angle A is congruent to measure of angle B, then angle will be equal to angle B.

To know more about the congruent angles, here

https://brainly.com/question/11949261

#SPJ4


Related Questions

A falcon was recorded travelling 100 metres in 0. 93 seconds.


What was its average speed


a in m/s to 3 significant figures

Answers

The average speed of the falcon to 3 significant figures will be 107.526 m/s

Distance travelled by falcon = 100 meters

Time taken by falcon to travel the distance = 0.93 seconds

As we have the formula for finding the average speed:

Average speed is a pace defined as a quantity divided by the time required to obtain that quantity. Meters per second is the SI unit of speed. The formula S = d/t is used to compute average speed, where S is the average speed, d is total distance, and t is total time.

Average speed can be find by simply dividing the total distance covered by falcon by the total time taken by him to cover the distance.

So, the value of the average speed will be = (100 m/0.93 sec) =107.526 m/s

For more questions on Average speed

https://brainly.com/question/24739297

#SPJ4

b. How should Kiran change the quantities that he used
so that his smoothie tastes
just like Clare's?

Answers

Answer:

Kiran's smoothie would be more chocolatey than Clare's. All ingredients are doubled, but there is an extra teaspoon of chocolate syrup in his smoothie.

Step-by-step explanation:

(aryan)

Using data from 1944 through 2000, the histogram shows the number of years that had a particular number of hurricanes reaching the East Coast of the U.S. For example, in 14 of those years there was exactly one hurricane each year that reached the East Coast of the U.S. What is the median number of hurricanes per year reaching the East Coast from 1944 through 2000

Answers

The median number of hurricanes per year reaching the East Coast from 1944 through 2000 is two.

What is median number?

Median number is the middle value of a set of numbers. It is the number found at the midpoint of a sorted data set, when all the numbers are arranged in numerical order from lowest to highest. The median number is used to determine the average of a set of numbers, and is often used to represent the central tendency of the data. To calculate the median, first arrange the numbers from lowest to highest, then locate the middle value. If there is an even number of values, the median is the average of the two middle numbers.

This is calculated by taking the middle value of the data set. In this case, there were 18 years with one hurricane, 16 years with two hurricanes, 13 years with three hurricanes, and six years with four hurricanes. Thus, the median number of hurricanes in this data set is two.

To know more about median number click-
https://brainly.com/question/1820596
#SPJ4

Solve the following system of equations: −2x + 3y = 7 y = 3x + 7 (−5, −1) (−2, 1) (−1, 4) (1, 3)

Answers

Answer:

(-2, 1)

Step-by-step explanation:

−2x + 3y = 7

y = 3x + 7

Since the second equation is already solved for y, we can use the substitution method. Substitute y of the first equation with 3x + 7 and solve for x.

-2x + 3y = 7

-2x + 3(3x + 7) = 7

-2x + 9x + 21 = 7

7x = -14

x = -2

Now substitute x in the second equation with -2 and solve for y.

y = 3x + 7

y = 3(-2) + 7

y = 1

Answer: (-2, 1)

f(x) and g(x) below, find the value of (fog)(-8).
f(x) = 3x²-3x - 8
g(x) = -x - 7

Answers

Therefore , the solution of the given problem of function comes out to be fog(-8)= 622.

Function: A Definition

A function is a relationship between a collection of sources or one product for each input. Simply described, a function is a group of equations linked to a single, distinct output. A declaration, rule, or law that specifies the relationship between two variables is known as a function in mathematics (the dependent variable). When a rules of this kind is employed, there is just one output. by photographer Alex Federspiel. This sentence provides a y=x2 example. Any input for x has only one output for y. Since x is indeed the input value, y is, by definition, a function of x.

Here,

Given :

f(x) = 3x²-3x - 8

g(x) = -x - 7

To find : fog)(-8)= ?

Thus,

We find

=> F(g(x)) => 3g(x)² -3g(x) - 8

=> f(-x-7) =3⋅(-x-7)²-3⋅(-x-7)-8

At x =-8

=> 3(-8-7)² - 3(-8-7)-8

=> 3(225)-3(15) -8

=> 675 - 45 -8

=> 622

Therefore , the solution of the given problem of function comes out to be fog(-8)= 622.

To know more about function visit :-

brainly.com/question/12431044

#SPJ1

Why 1 [infinity] is not defined?

Answers

1 [infinity] is not defined because infinity is an abstract concept and cannot be used in the same way as numeric values. Infinity is not a number or a numerical value, and therefore it cannot be used in equations or arithmetic operations.

1. Infinity is an abstract concept and cannot be used in the same way as numeric values.

2. Infinity is not a number or a numerical value.

3. Therefore, it cannot be used in equations or arithmetic operations.

4. Therefore, 1 [infinity] is not defined.

1 [infinity] is not defined because infinity is an abstract concept and cannot be used in the same way as numeric values. Infinity is not a number or a numerical value, and therefore it cannot be used in equations or arithmetic operations.

Learn more about number here

https://brainly.com/question/10547079

#SPJ4

A club consisting of 11 men and 12 women needs to choose a committee from among its members so that the number of women on the committee is one more than the number of men on the committee. The committee could have from 1 to 23 members. How many committees can be formed

Answers

The total number of such committees that may be created, divided by the prime numbers that divide N, is 81.

What is the  prime numbers that divide N?

In the scenario presented in this question, a club with 11 men and 12 women must elect a committee from among its members in order for there to be one more woman on the committee than males.

The committee might consist of only 1 person or up to 23 people.

Let N represent the maximum possible number of such committees.

The total number of prime numbers that divide N must be determined.

The number of ladies chosen will be m.

The number of guys who were not chosen would thus be:

11 - (m - 1) = 12 - m

Each pairing of men who were not chosen and women who were chosen represents a committee choice.

Since selecting 12 people from a total of 23 would result in m women and 12 - m males, the committee chose:

making use of a combination formula

N = 23C12

N = 23!/12!(23 - 12)!

N = 1352078

These are the main components of 1352078: 2, 7, 13, 17, 19, and 23.

Consequently, the prime number total would be,

2 + 7 + 13 + 17 + 19 + 23 = 81

The total of the prime numbers is thus 81.

The complete question is:

A club consisting of 11 men and 12 women needs to choose a committee from among its members so that the number of women on the committee is one more than the number of men on the committee. The committee could have as few as 1 member or as many as 23 members. Let N be the number of such committees that can be formed. Find the sum of the prime numbers that divide N.

To learn more about prime numbers refer to:

https://brainly.com/question/30011920

#SPJ4

What is a 100 sided shape called?

Answers

Answer:

hectogon or hecatontagon or 100-gon

Step-by-step explanation:

It is a hundred - sided polygon ( shape ). The sum of all hectogon's interior angles are 17640 degrees.

Answer:

Regular exercise has been shown to aid in the prevention and management of noncommunicable illnesses such as diabetes, heart disease, stroke, and a number of malignancies. Additionally, it lowers blood pressure, supports healthy body weight, and enhances well-being through enhancing mental health.

Step-by-step explanation:

What would be the explicit equation for the sequence 3, 9, 27, 81 

Answers

Answer:

times it by

Step-by-step explanation:

3×3=9, 9×3=27, 27×3=81, 81×3=243

sorry if this isn't the answer your looking for.

Answer: multiply by 3 each time
3*3=9
3*9=27
3*27=81

Marcus is shopping for pants. The available styles are boot cut (B), skinny (S), and relaxed fit (R). Make an organized list to show all possible outcomes if he buts two new pair of pants.

Answers

There are 9 possible outcomes  to buy two new pair of pants.

What is combination?

Combinations are mathematical operations that count the number of potential configurations for a set of elements when the order of the selection is irrelevant. You can choose the components of combos in any order.

Given:

we have the following Styles

Boot cut (B), Skinny (S), and Relaxed fit (R).

So, if her buys two paints then the possible combination he can take as

Boot cut x Boot cut

Skinny x Skinny

Relaxed fit x Relaxed fit

Boot cut x Skinny

Boot cut  x Relaxed fit

Skinny x Relaxed fit

Skinny x Boot cut

Relaxed fit x Boot cut

Relaxed fit x Skinny

Hence, there are 9 possible ways.

Learn more about Combination here:

https://brainly.com/question/13387529

#SPJ1

If P is the midpoint of ST, SP = x + 4, and ST = 4x, determine the length of ST.

Answers

The length of ST is 16 units.

What is Line segment?

A line segment is a part of line having two endpoints and it is bounded by two distinct end points and contain every point on the line that is between its endpoint.

We have to given that;

Point P is the midpoint of ST.

SP = x + 4, and ST = 4x

Since, P is the midpoint of ST.

Hence, We get;

⇒ SP = PT

And, ST = 2 SP

Substitute all values we get;

⇒ 4x = 2 (x + 4)

⇒ 4x = 2x + 8

⇒ 4x - 2x = 8

⇒ 2x = 8

⇒ x = 4

Thus, The value of ST = 4x

                                  = 4×4

                                  = 16

Learn more about the line segment visit:

https://brainly.com/question/280216

#SPJ1

PLEASE HELP ASAP





Which is the following is not an attribute of a right rectangular prism?


1 apex

12 edges

rectangular faces

square bases

Answers

Square bases is not an attribute of a right rectangular prism. Option D

What is a right rectangular prism?

A right rectangular prism can be described as a three-dimensional solid shape with 6 faces, 12 edges, and 8 vertices.

It is sometimes called a cuboid.

All the faces of the right rectangular prism are rectangular in shape.

Examples of  right rectangular prisms are books, bricks and aquarium.

Some properties of a right rectangular prism include;

There are 12 edges There are 8 verticesAll the faces are rectanglesAll the angles formed at the vertices are of 90 degrees

These prisms have no square base(s)

Hence, the right option is square bases.

Learn more about right rectangular prisms here:

https://brainly.com/question/9243999

#SPJ1

The admission fee to the carnival is $4. 50 for children and $7. 25 for adults. If a large group purchased 28 tickets and paid $159 how many adult tickets did they purchase?

Answers

The number of adult tickets purchased is 12.

What is the arithmetic operation?

The four fundamental operations of arithmetic are addition, subtraction, multiplication, and division of two or more quantities. Included in them is the study of numbers, especially the order of operations, which is important for all other areas of mathematics, including algebra, data management, and geometry. The rules of arithmetic operations are required in order to answer the problem.

Let no. of adult = x

No. of children = 28 -x

So, 7. 25 * x + (28 - x) *4. 50  = 159

⇒2.75x +126=159

⇒2.75x =33

⇒x=33/2.75

x=12

Hence, the number of adult tickets purchased is 12.

Learn more about arithmetic, by the following link.

https://brainly.com/question/13181427

#SPJ4

the graph represents the piecewise function

Answers

The piecewise function represented by the graph is:

f(x) = 2x when -3 ≤ x < 0

f(x) = 3 When 1/2 < x < 3/2

What is a function?

function is a relationship between inputs where each input is related to exactly one output.

Example:

f(x) = 2x + 1

f(1) = 2 + 1 = 3

f(2) = 2 x 2 + 1 = 4 + 1 = 5

The outputs of the functions are 3 and 5

The inputs of the function are 1 and 2.

We have,

Let f(x) = y = mx + c

From the graph, when -3 ≤ x < 0.

(-3, -6) and (-1, -2)

m = (-2 + 6) / (-1 + 3) = 4 / 2 = 2

(-1, -2) = (x, y)

-2 = 2 x (-1) + c

-2 = -2 + c

c = 0

f(x) = 2x

From the graph, When 1/2 < x < 3/2.

It is a horizontal line of y = 3.

f(x) = 3

Thus,

f(x) = 2x when -3 ≤ x < 0

f(x) = 3 When 1/2 < x < 3/2

Learn more about functions here:

https://brainly.com/question/28533782

#SPJ1

What is Polya's 4 step method?

Answers

Polya's 4 step method is a process designed to help problem solvers become better at tackling difficult problems.

It was developed by the mathematician George Polya in 1945 and consists of four basic steps:

1. Understand the problem: The first step is to read and understand the problem. This means understanding the problem and identifying what information is given and what is required.

2. Devise a plan: The second step is to identify a strategy or plan of attack. This involves breaking the problem down into smaller parts, identifying the key elements, and using logical reasoning to identify a solution.

3. Carry out the plan: The third step is to execute the plan. This involves following the steps of the plan in order to solve the problem.

4. Look back: The fourth and final step is to take a look back at the solution and evaluate it. This involves checking to make sure the solution is correct and making any necessary adjustments.

Learn more about Polya's method here:

https://brainly.com/question/21689931

#SPJ4

Give three solutions of the inequality
6 - 11x < 61

Answers

Answer:

x > -5

( -5, ∞)

Step-by-step explanation:

6 - 11x < 61

Subtract 6 from both sides

-11x < 55

Divided both side by -11. When divided by a negative number the < switch to >

So, the answer is

x > -5

Or

(-5,  ∞)

Calculate the size of the angle marked 'x' in the pentagon shown below. Give your answer in degrees (°).

Answers

Answer: x= 96 degrees

Step-by-step explanation:

It we figure out the rest of the angles they would be (the outside ans inside angles are straight angles):

123+153+49+119= 444

Since the total of angles in a pentagon = 540 degrees

540-444 96 degrees

The measure of an angle x will be equal to 96°.

What is the Pentagon?

A Pentagon is a two-dimensional geometric shape that consists of five straight sides and five angles. It is a type of polygon, which is a closed shape with three or more sides.

The Pentagon is often used to represent the shape of buildings, monuments, and symbols in architecture and design.

If we figure out the rest of the angles they would be (the outside and inside angles are straight angles):

123+153+49+119= 444

Since the total of angles in a pentagon = 540 degrees

540-444= 96°

Therefore, the measure of an angle x will be equal to 96°.

To know more about the Pentagon follow

https://brainly.com/question/30733186

#SPJ2

Given the answer for part D, write an expression that will tell you the direction the robot is going if, in the course of its journey, it turns left 21 times and turns right 22 times. Does the order the robot makes the turns in matter for the purpose of knowing the direction it is finally facing?
The answer to part D is in the picture...

Answers

The robot is moving in the direction indicated by the formula ((i)²¹*(-i)²²)d, or in the beginning direction.

What is complex plain?

When a line is rotated in the complex plane either clockwise or anticlockwise, it is multiplied by -i or I depending on which direction the line is rotated. The direction vector is rotated 90 degrees in the clockwise direction to make a right turn. It is the same as increasing the complex number by -i.

A left turn is represented as a 90° anticlockwise rotation of the direction vector. It is comparable to increasing the complex number by i.

There are 21 left turns of the robot. The direction vector will now be di21. Once it has been rotated 22 times to the right, the direction vector will now be  (di²¹)*(-i)²²=d((i)²¹*(-i)²²).

Consequently, the robot's current direction is:

d((i)²¹*(-i)²²)= di²¹⁺²²=di⁴³= d(-i)= -di

No matter what, the direction will always be the same, hence the sequence is irrelevant.

Hence, the equation that will indicate where the robot is moving is  ((i)²¹*(-i)²²)d, or in the starting direction. Furthermore, the final direction it faces does not depend on the order.

Learn more about complex plain, by the following link.

https://brainly.com/question/16835201

#SPJ4

find the height of the two pillar with same height​

Answers

Answer:

ED = AB = 25√3

Step-by-step explanation:

Let ED = AB = X

DC = 100 - BC

Tan 60 = [tex]\frac{\sqrt{3} }{1} = \frac{ED}{DC}[/tex] = [tex]\frac{X}{100-BC}[/tex]

[tex]\sqrt{3} (100-BC) = X[/tex]

Tan 30 = [tex]\frac{1}{\sqrt{3} } = \frac{AB}{BC} = \frac{X}{BC}[/tex]

[tex]\frac{BC}{\sqrt{3} } = X[/tex]

[tex]\sqrt{3} (100-BC) = \frac{BC}{\sqrt{3} } \\3(100-BC) = BC\\300 - 3BC = BC\\300 = BC + 3BC\\300 = 4BC \\75 = BC[/tex]

X = [tex]\frac{BC}{\sqrt{3} } = \frac{75}{\sqrt{3} }\\[/tex]

X = 25√3

prove that cos x = sin x cot x.

Answers

Answer:

can u give me the brainliest

the answer is in the attached documents

What is the value of x if the points A 4 3 and B x 5 are on the circle with Centre O 2 3?

Answers

The value of x with points A ( 4, 3 ) and B ( x ,5 ) on the circle and O(2,3) as centre is equal to 2.

As given in the question,

Points A(4,3 ) and B ( x, 5 ) are on the circle.

Centre of the circle is represented by O( 2,3)

OA and OB represents the radius of the circle.

This implies

OA = OB

Using distance formula we get,

√ ( 2 - 4 )² + ( 3 - 3 )² = √ ( 2 - x )² + ( 3 - 5 )²

⇒ √ 4 + 0 = √ 4 - 4x + x² + 4

⇒4 =  4 - 4x + x² + 4

⇒4 - 4x + x² = 0

⇒ ( x - 2 )² = 0

⇒ x - 2 = 0

⇒ x = 2

Therefore, the value of x for the given point of the circle is equal to 2.

The above question is incomplete , the complete question is :

What is the value of x , if the points A(4, 3) & B(x, 5) are on the circles with Centre O(2, 3), find the value of x.

Learn more about circle here

brainly.com/question/29142813

#SPJ4

.A car is traveling on a small highway and is either going 55 miles per hour or 35 miles per hour, depending on the speed limits, until it reaches its destination 200 miles away. Letting x represent the amount of time in hours that the car is going 55 miles per hour, and y being the time in hours that the car is going 35 miles per hour, an equation describing the relationship is: 55x+35y=200


A: If the car spends 2.5 hours going 35 miles per hour on the trip, how long does it spend going 55 miles per hour?

B: If the car spends 3 hours going 55 miles per hour on the trip, how long does it spend going 35 miles per hour?

C: If the car spends no time going 35 miles per hour, how long would the trip take? Explain your reasoning.

Answers

Answer:

This equation can be used to determine how long the car was traveling at each speed. You can solve for one of the variables by using algebraic techniques. For example, you could subtract 35y from both sides to get:

55x = 200 - 35y

Then divide both sides by 55 to get:

x = (200 - 35y)/55

This equation shows that the value of x (the time spent traveling at 55 mph) is determined by the value of y (the time spent traveling at 35 mph). If you know the value of y, you can use this equation to calculate the value of x.

To find the values of x and y that satisfy the equation, you will need to use additional information. For example, you could set up a system of equations and solve for x and y using techniques such as substitution or elimination.

Step-by-step explanation:

The equation can be used to determine how long the car was traveling at each speed

According to the 2000 U.S. Census, 7 out of every 25 homes are heated by electricity. At this rate, predict how many homes in a community of 12,000 would be heated by electricity.a.6,725 homesb.3,100 homesc.3,360 homesd.4,020 homes

Answers

In 7/25 households, electricity is used for heating. By multiplying by 20,000. You'll receive 3,360. In light of this, the 2000 census indicates that 3,360 out of 12,000 houses use electricity for heating.

How many homes use electricity for heating in U.S. Census?

Because we've already done it 7 out of every 25 residences are heated by electricity, according to the 2000 U.S. census.At this rate, we must determine how many residences in a 12000-person neighbourhood would be heated by electricity.Let x be the number of homes in a neighbourhood that would be heated by electricity.There being a direct variation

The result is that , A neighbourhood of 12000 people with 3360 dwellings that use electricity for heating. In 7/25 households, electricity is used for heating. By multiplying by 20,000. You'll receive 3,360. According to the 2000 Census, 3,360 dwellings out of 12,000 are heated by electricity.

[tex]$\frac{7}{25}=\frac{x}{12000}$[/tex]

[tex]$x={7 \times 12000}/{25}$$x=3360$[/tex]

As a result, 3360 households out of the 12000 in the community use electricity for heating.

To learn more about U.S. Census refer to :

https://brainly.com/question/2141843

#SPJ4

On a coordinate plane, a line goes through (negative 12, negative 2) and (0, negative 4). A point is at (0, 6).Which point is on the line that passes through (0, 6) and is parallel to the given line? (–12, 8) (–6, 6) (2, 8) (6, 0)

Answers

Point (-12 , 8) is on the line that passes through (0, 6) and is parallel to the given line.

What is Equation of line?

The equation of line in point-slope form passing through the points

(x₁ , y₁) and (x₂, y₂) with slope m is defined as;

⇒ y - y₁ = m (x - x₁)

Where, m = (y₂ - y₁) / (x₂ - x₁)

Since, We know that;

Parallel lines have same slopes and different y-intercepts

The formula of the slope of a line which passes through points (x₁ , y₁) and (x₂, y₂) is,

⇒  m = (y₂ - y₁) / (x₂ - x₁)

Since, The given line passes through points (-12 , -2) and (0 , -4)

Hence, Use the formula of the slope above to find the slope of the given line as;

⇒ m = (y₂ - y₁) / (x₂ - x₁)

⇒ m = (-4 - (-2)) / (0 - (-12))

⇒ m = (-2) / 12

⇒ m = - 1/6

Since, The two lines are parallel.

Hence, Their slopes are equal

So, The slope of the parallel line = - 1/6

Here, The parallel line passes through point (0 , 6)

Since, The form of the linear equation is y = mx + b, where m is the slope and b is the y-intercept.

⇒ m = -1/6 and b = 6

Hence, The equation of the parallel line is,

⇒ y =  x + 6

Now, Let us check which point is on the line by substitute the x in the equation by the x-coordinate of each point to find y, if y is equal the y-coordinate of the point, then the point is on the line as;

Point (-12 , 8)

Substitute x = -12 and y = 8

⇒ y =  (-12) + 6

⇒ y = 2 + 6 = 8

Clearly, The value of y is equal the y-coordinate of the point.

So, Point (-12 , 8) is on the line.

Learn more about the equation of line visit:

https://brainly.com/question/18831322

#SPJ1

Evaluate the gradient of f(x, y, z) = log(x2 + y2 + z2) at (1, 0, 1)

Answers

The gradient of f(x, y, z) = log(x2 + y2 + z2) at (1, 0, 1) is (2/1, 0, 2/1)

This gradient is a vector that points in the direction of the greatest rate of increase of the function f(x, y, z) = log(x2 + y2 + z2). To evaluate this gradient, the partial derivatives of the function with respect to x, y, and z must be calculated.

The partial derivative of the function with respect to x is (2x / (x2 + y2 + z2)). When x is 1, this simplifies to 2/1. Similarly, the partial derivative with respect to y is 0, and the partial derivative with respect to z is (2z / (x2 + y2 + z2)), which simplifies to 2/1 when z is 1. Therefore, the gradient of f(x, y, z) = log(x2 + y2 + z2) at (1, 0, 1) is (2/1, 0, 2/1).

Learn more about Gradient here:

https://brainly.com/question/30120866

#SPJ4

What kind of triangle has 25 90 65?

Answers

25-90-65 is a right-angle triangle because one of the angles is 90°.

A right-angle triangle or orthogonal triangle is a triangle in which one angle is a right angle (that is, a 90-degree angle), and two sides are perpendicular. The relationship between the sides and angles of the right triangle is the basis for trigonometry.

The hypotenuse is the facing side to the right angle. Legs are the adjoining sides to the right angle.

Properties of right angle triangle:

The right angle is the largest angle in a right-angle triangle.The hypotenuse, the opposite of the right angle, is the longest side.There are no obtuse angles in a right-angle triangle.

Read more about right-angle triangles:

https://brainly.com/question/64787

#SPJ4

How many 1/3 cubes does it take to fill a box with a width of 2 2/3 inches length 3 1/3 inches and height 2 1/3

Answers

It takes 560 of 1/3 inch cubes to fill a box with width 2 2/3 inches, length 3 1/3 inches, and height 2 1/3 inches.

In Math's or in Geometry, a Cube is a solid three-dimensional figure, which has 6 square faces, 8 vertices and 12 edges.

It is also said to be a regular hexahedron.

We will convert the dimensions of the given box into simple fractions for easy calculations.

The dimensions of the given box:

Length = 3 + 1/3 inches = 10/3 inches

Breadth = 2 + 2/3 inches = 8/3 inches

Height = 2 + 1/3 inches = 7/3 inches

The side of each cube = 1/3 inches

Therefore,

The no. of cubes required to fill the box is,

= (volume of the box) / (volume of each 1/3 inch cube)

= ( 10/3 * 8/3 * 7/3 ) / (1/3)^2

= 10 * 8 * 7

= 560

Thus, it takes 560 of 1/3 inch cubes to fill a box with width 2 2/3 inches, length 3 1/3 inches, and height 2 1/3 inches.

To learn more about cubes visit : brainly.com/question/29372770

#SPJ4

find the values of x, z and y

Answers

The values of x is 82° , z is 98°and y is 69°. in the given triangle.

Is the sum of all triangle angles always 180 degrees?

Because one outside angle of a triangle is equal to the total of the other two angles in the triangle, the angles of a triangle add up to 180 degrees. In other words, the third angle must join with the other two angles in the triangle to generate a 180-degree angle (the two angles that add up to form the outside angle).

we know that the sum of angle of triangle = 180

so,

x+ 60 +38=180

x=180-8

x=82°

and also

x+z = 180

z= 98°

for value of y

13 + 98 +y = 180

y= 69°

To know more about angle of triangle  visit:-

brainly.com/question/28793979

#SPJ1

hi everyone help my math pls
show ur solutions ​

Answers

On solving the provided question, we can say that - the area of the provided circle is A = [tex]113.04 cm^2[/tex]

What is circle?

Every point in the plane that is a certain distance away from a certain point forms a circle (center). It is, thus, a curve formed by points moving in the plane at a fixed distance from a point. At every angle, it is also rotationally symmetric about the center. A circle is a closed two-dimensional object where every pair of points in the plane are equally spaced out from the "center." A line that goes through the circle creates a specular symmetry line. At every angle, it is also rotationally symmetric about the center.

here,

radius, r = 6 cm

Area of circle = [tex]\pi X r^2 = 3.14 X 6 X 6 \\[/tex]

A = [tex]113.04 cm^2[/tex]

To know more about circle visit:

https://brainly.com/question/29142813

#SPJ1

upper and lower bounds question

Answers

The upper bound and lower bound of a using the equation will result to

upper bound value 136015.499

lower bound value 136014.500

How to find the upper bound and lower bound values

Evaluating the given expression a = b + 2c

where

b = 15 to 2 significant figures

c = 68 000 to 2 significant figures

solving for a

a = b + 2c

a = 15 + 2(68000)

a = 136015

The bound values

upper bond value is the largest possible vale of a

upper bond value = 136015.499

lower bound value is the lowest possible value of a

lower bound value = 136014.500

Learn more about bound values at:

https://brainly.com/question/26155120

#SPJ1

Other Questions
Notice the number of edge tiles (column 3) is equal to two less than the number of tiles on one side (column 1) multiplied by four. Knowing this relationship, if we built a 10 by 10 square mirror, write a numerical expression that would help us calculate the number of edge tiles we would need. Breaking the Confederate spirit and forcing the social reformation of the South proved Idealistic, and Northerners eventually gave up or lost interest.The troops were removed following the ______and the South rose again... re-implementing a racist culture with segregation anddisenfranchisement. why are ventilations for a room put near the roof and not near the floor Which of these statements is true?c. The best fossils are formed during a cataclysmic event.a. Younger strata are usually nearer the surface.d. Rock strata are formed by volcanic activity.e. Waters ability to erode rock helps to explain how fossils are formed.b. More recent fossils are in the deepest An investor purchased a corporate bond for 97 3/8. If the bond is sold for 99 3/8, the investor has a profit of Are the following statements true or false? Justify your answers. Complete parts (a) through (c) below. (This is a reading assessment question. Be certain of your answer because you only get one attempt on this question.) (a) In statistics, results are always reported with 100 % certainty. Choose the correct answer below A. False. In statistics, results are always reported with 100 % certainty only when discrete variables are measured. Results cannot be reported with 100 % certainty when continuous variables are measured. B. True. Statistics is the science of collecting, organizing, summarizing, and analyzing information to draw conclusions or answer questions. Since it is a science, results are always reported with 100 % certainty. C. True. Statistics would not be useful if they were not presented with 100 % certainty . Statistics is a form of mathematics, which means that as long as correct procedures are used, all results must be correct. D. False. In statistics, results are not reported with 100% certainty. Because statistical studies draw on samples, and because there is variation within groups, results cannot be reported with 100 % certainty. Why does the author describe the symbols found on American money in paragraph 3? Why is group 17 more reactive than Group 16? HELP!! Find the inverse of the matrix What is an example of an irregular or variable expense? A9. How does the writer try to persuade us that hedgehogs should be protected? How do you solve 7x 5y? What is the main theory of evolution? which form of treatment is used to promote the healing process by dilating blood vessels which allows for more circulation to occur in the affected area. Thirst is a reliable indicator of how much water should be consumed to replenish body fluids during illness.Choose matching term1. Compared to whole grain products, refined grains are ________ in trace minerals.2 .Which of the following is not true about the regulation of fluid in the body?3. Minerals are categorized based on the amount we need per day. Which of the following statements is correct?4. The amount of water in each body compartment mainly is controlled by For an entire species to survive, it must allow for new generations to maintain the population. This is the challenge of ___________. You have a small globe, which is mounted so that it can spin on the polar axis and can be spun about a horizontal axis (so that the south pole can be on top).Give the globe a quick spin about the polar axis, and then, before it stops, give it a spin about the horizontal axis. Are there any points on the globe that are at rest?(A) There are two points, fixed on the globe, that are at rest,(B) There are two points that are instantaneously at rest, but these two points move around the globe in an apparently random fashion,(C) At some times two points are instantaneously at rest,(D) There are no points at rest until the globe stops spinning What is statistics. Explain? please help!Use the Fundamental Counting Principle to determine how many kinds of teddy bears you can build if you must choose one of each category.1) Choose the bear.2) Choose an accessory. 3) Choose sound option. Eve has been keeping an eye on an emerald ring she likes. Its original price was $1,730. Afterbeing marked down for a clearance sale, it is now $1,038. By what percent has the price of thering gone down?